Difference between revisions of "2008 AMC 8 Problems/Problem 9"

(Created page with "==Problem 9== In <math>2005</math> Tycoon Tammy invested <math>100</math> dollars for two years. During the the first year her investment suffered a <math>15\%</math> loss, but d...")
 
Line 1: Line 1:
==Problem 9==
+
==Problem==
 
In <math>2005</math> Tycoon Tammy invested <math>100</math> dollars for two years. During the the first year
 
In <math>2005</math> Tycoon Tammy invested <math>100</math> dollars for two years. During the the first year
 
her investment suffered a <math>15\%</math> loss, but during the second year the remaining
 
her investment suffered a <math>15\%</math> loss, but during the second year the remaining

Revision as of 13:04, 9 December 2012

Problem

In $2005$ Tycoon Tammy invested $100$ dollars for two years. During the the first year her investment suffered a $15\%$ loss, but during the second year the remaining investment showed a $20\%$ gain. Over the two-year period, what was the change in Tammy's investment?

$\textbf{(A)}\  5\%\text{ loss}\qquad \textbf{(B)}\ 2\%\text{ loss}\qquad \textbf{(C)}\ 1\%\text{ gain}\qquad \textbf{(D)}\ 2\% \text{ gain} \qquad \textbf{(E)}\  5\%\text{ gain}$

See Also

2008 AMC 8 (ProblemsAnswer KeyResources)
Preceded by
Problem 8
Followed by
Problem 10
1 2 3 4 5 6 7 8 9 10 11 12 13 14 15 16 17 18 19 20 21 22 23 24 25
All AJHSME/AMC 8 Problems and Solutions